Mostrar Mensajes

Esta sección te permite ver todos los posts escritos por este usuario. Ten en cuenta que sólo puedes ver los posts escritos en zonas a las que tienes acceso en este momento.

Mensajes - Luis Fuentes

Páginas: [1] 2 3 4 ... 2744
1
Hola

Luis fuentes gracias por responder, anhelaba que tomarás en cuenta mi pregunta, ahora la pregunta que te hago es la siguiente, si he planteado bien las ecuaciones? De ser así como doy respuesta al ejercicio? Que puedo decir al respecto?

Si te refieres a las preguntas originales:

Un comerciante acude al mercado popular a comprar naranjas con \( 5000 $ \). Le ofrecen dos tipos de naranjas: las de tipo A a \( 5 $ \) el kg. y las de tipo B a \( 8 $ \). el kg. Sabiendo que sólo dispone de su camioneta con espacio para transportar \( 700 kg \). de naranjas como máximo y que piensa vender el kg. de naranjas tipo A a Bs. \( 6 $ \). y el kg. de tipo B a \( 9 $ \).
¿Cuántos kg. de naranjas de cada tipo deberá comprar para obtener máximo beneficio? ¿Cuál será ese beneficio máximo?

  a)   ¿Este problema se puede plantear  como un problema de Programación Lineal? Me pueden Explicar ¿Por qué?                           
  b)  cómo se  Formula matemáticamente este problema de programación lineal?

 a) Si, se puede plantear como un problema de programación lineal porque tanto las restricciones como la función objetivo son lineales.
 b) Se formula como se ha explicado en el hilo.

 Si te refieres la respuesta en cuanto al óptimo sería: el óptimo es un beneficio de \( 700 \)$ que se alcanza comprando \( x \) de tipo \( A \) y \( 700-x \) de tipo \( B \) para cualquier valor de \( x\in [200,700] \).

Saludos.

2
Foro general / Re: Encontré un número perfecto impar
« en: Hoy a las 02:07 pm »
Hola

Me declaro un ignorante en este tema y creo que no puedo aportar nada; no obstante, me parece que del hecho de que el \( 1 \) carezca de divisores propios se podría deducir tanto que la suma de ellos sea \( 0 \) como que sea \( 1 \) o cualquier otro valor. Este asunto me recuerda a un comentario que ha hecho Masacroso en alguno de sus mensajes aludiendo a la verdad vacía. Si el conjunto de divisores propios del \( 1 \) es vacío ¿no se podría sostener que no hay propiedad alguna que no cumplan sus elementos? Es solo una pregunta y un interés en conocer otras opiniones.

Lo que se tiene es que la proposición:

\( x\in \emptyset\quad \Rightarrow{}\quad x \) cumple \( P(x) \)  (*)

es cierta independientemente de la propiedad \( P(x) \) que pongas.

Pero no se muy bien que tiene que ver con este asunto, aquí en todo caso está el matiz de como interpretar:

\( \displaystyle\sum_{a\in A}a \)

Cuando \( A \) es el conjunto vacío.  Asignarle cualquier valor a esa suma no tiene nada que ver con una proposición del tipo (*).

Si \( A \) es el vacío se le suele asignar por definición a ese sumatorio el valor cero; tiene bastante sentido porque uno no está sumando nada y además digamos que las fórmulas que involucran sumatorios suelen funcionar bien con ese convenio. Pero no deja de ser un convenio.

Por ejemplo en el caso de productorios el convenio es que si no se multiplica nada, se le asigne el valor \( 1 \) (el neutro del producto). Y en general si se hace un análogo con cualquier operación de grupo, cuando el conjunto de elementos que opera es vacío el convenio suele ser asignarle como resultado el elemento neutro.

Saludos.


3
Hola

Tanto en  el punto b, como en el c) los resultados se repiten, estaré haciendo malos cálculos? Cómo interpreto estos resultados? Puedo decir que según el punto b) para obtener el máximo beneficio debo de vender \( 200 kg \) de naranjas de tipo a y \( 500kg \) de tipo  b? Y también podría obtener el máximo beneficio vendiendo \( 700kg \) naranjas de de tipo A? Ojalá me pueden ayudar porque estoy confundida con estos resultados. Porque se repiten.

Eso significa que el óptimo no se alcanza en un único punto, y de hecho valdría cualquier punto del segmento que une \( (200,500) \) con \( (700,0) \)

El fenómeno ocurre porque si te fijas la función a optimizar \( f(x,y)=x+y \) coincide con una de las restricciones \( x+y\leq 700 \), es decir, todos los puntos de la forma \( f(x,y)=700 \) cumplen esa restricción.

Saludos.

4
Foro general / Re: Encontré un número perfecto impar
« en: Hoy a las 01:55 pm »
Hola

 Antes de nada dainzafa, Fernando Revilla te está insistiendo mucho en que lo que estás haciendo es cambiar la definición de lo que la comunidad matemática entiende por número perfecto (o divisor propio), y no hay debate al respecto:

- Con las definiciones usuales que se manejan en matemáticas de número perfecto. El \( 1 \) NO es perfecto.

Spoiler
Entendiendo que trabajamos en el conjunto de enteros positivos:

 i) Un número \( a \) es divisor de \( b \) si existe otro entero \( c \) tal que \( ac=b \).
 ii) Un número \( a \) es divisor propio de \( b \) si es divisor de \( b \) y además \( a\neq b \).
iii) Un número \( a \) es perfecto si es igual a la suma de todos sus divisores propios.

 ¿Aceptando esas definiciones estás de acuerdo en que el \( 1 \) no es perfecto?.

Ej:

 - Divisores de \( 1=\{1\} \). Divisores propios de \( 1 \): ninguno (porque \( 1=1 \)). Suma de los divisores propios: \( 0\neq 1 \) (no hay nada que sumar). Por tanto el \( 1 \) no es perfecto.
 - Divisores de \( 2=\{1,2\} \). Divisores propios de \( 2=\{1\} \). Suma de los divisores propios: cero: \( 1\neq 2 \). Por tanto el \( 2 \) no es perfecto.
 - Divisores de \( 3=\{1,3\} \). Divisores propios de \( 3=\{1\} \). Suma de los divisores propios: cero: \( 1\neq 3 \). Por tanto el \( 3 \) no es perfecto.
 - Divisores de \( 4=\{1,2,4\} \). Divisores propios de \( 4=\{1,2\} \). Suma de los divisores propios: cero: \( 1+2=3\neq 4 \). Por tanto el \( 4 \) no es perfecto.
 - Divisores de \( 5=\{1,5\} \). Divisores propios de \( 5=\{1\} \). Suma de los divisores propios: cero: \( 1=1\neq5 \). Por tanto el \( 5 \) no es perfecto.
 - Divisores de \( 6=\{1,2,3,6\} \). Divisores propios de \( 6=\{1,2,3\} \). Suma de los divisores propios: cero: \( 1+2+3=6  \). ¡Bingo! Por tanto el \( 6 \) SI es perfecto.
[cerrar]

- Tu modificas esas definiciones considerando que el \( 1 \) si es divisor propio de si mismo y entonces \( 1=1 \) y si es perfecto. Bien. Pero en ese caso no es que tu hayas descubierto un número perfecto nuevo; no es que nadie se hubiese dado cuenta hasta ahora de que el \( 1 \) era perfecto, si no que para ti número perfecto es algo sutilmente distinto a lo que la matemática usual llama número perfecto y por eso consideras al uno como tal. ¿De acuerdo en esto?.

 Entonces, el debate ahí no tiene demasiado interés.

 Tendría interés si pudieses probar que no hay números perfectos impares mayores que el \( 1 \) o que dieses un ejemplo de número perfecto impar mayor que \( 1 \)..

 Afirmas que SI has probado que hay números perfectos impares, pero sinceramente no le veo sentido a tu argumento ya te he explicado porqué y no me has contestado explícitamente a mis críticas. Rescato lo que puse al respecto:

1- Un número perfecto es un número triangular, lo que es igual a la sumatoria desde k = 1 hasta n de k = n(n+1)/2

Cada uno de esos valores de k es un factor del número perfecto.

desde k = 2 en adelante, es imposible encontrar un número perfecto impar debido a que el 2 es un factor para cualquier número perfecto. Lo que quiere decir que todos los números perfectos son divisibles por 2, por lo que TODOS LOS NÚMEROS PERFECTOS MAYORES A 1, SON PARES.


No acabo de entender lo que haces. En primer lugar está demostrado que todo número perfecto PAR es triangula; pero no se sabe si todo número perfecto impar (en caso de que existiese) es triangular.

En segundo lugar un número triangular es de la forma \( n(n+1)/2 \) y ese cociente en principio pude ser par o impar. Así que incluso usando (sólo) que un número perfecto es triangular de ahí no se deduce que sea impar.

¿Algo qué decir sobre la frase subrayada en rojo?


Citar
Además no se de donde sacaste ese concepto de que N tiene que ser igual a la sumatoria de los divisores propios y ademas igual a la productoria de los divisores propios

Efectivamente no sé porqué Richard trajo a colación el productorio porque aquí no influye para nada; no interviene en la definición del número perfecto. Creo que se confundió.

Citar
Ese es el camino, no una convención matematicamente aceptada para hacer lo que no pudieron hacer con un razonamiento.

La RAE define:
“Primo: Del lat. primus.
1. adj. primero.
Sin.: primero, inicial.”

Si vas a usar la RAE para revisar la definición de primo mal vamos... Porque primo en matemáticas no significa primero. En ese caso sólo habría un primo, el primer elemento de algún conjunto (si es los enteros positivos el \( 1 \)). Pero eso no tiene nada que ver con la definición matemática de primo: un número primo es un número entero mayor que \( 1 \) que no puede ponerse como producto de dos números enteros positivos distintos del uno; o equivalentemente número entero mayor que uno que sólo es divisible por si mismo y por la unidad; o equivalentemente un número entero positivo que tiene exactamente dos divisiores.

Citar
Eso justifica al primer número natural, como el Número Primo por excelencia... Y ahora comproba todo lo que dije antes:

Entonces justificar la consideración de uno como primo por esa definición de la RAE no tiene sentido alguno.

En fin...

Saludos.

P.D. Nadie de burla de ti.

5
Hola

Chat GPT: NO es fiable en absoluto para cuestiones sobre matemáticas.
En nada es fiable, pero el planteo lo hizo bien ¿no? Erró al hacer las cuentas nomás.

Si; pero no es fiable insisto. Ya han salido ejemplos en el foro de que "liándolo" da por buenos razonamientos absurdos.

Citar
Citar
delmar ha explicado el razonamiento.
Lo siento, incluso ahora me cuesta entenderlo.

Cuando no entiendas algo detalla poco a poco la duda concreta.

Saludos.

6
Análisis Matemático / Re: Binomial Sum
« en: Hoy a las 10:29 am »
Hola

It does not have a very simplified solution

What about this?

\( \displaystyle \sum^{n}_{k=1}\frac{\binom{n+k}{k}}{\binom{n}{k}}\frac{2k-1}{n+k}=\displaystyle\binom{2n}{n}-1 \)

But, for now I don't know how to prove it

Best regards:

7
Hola

Los libros de matemáticas son una desilusión.

Bueno a veces es difícil evitar que se cuele alguna errata, aunque en este caso me choca un tanto las que se refieren a las definiciones de dependencia e independencia que se acercan más a un error conceptual que a un typo.

También, como restaurantes, hay libros mejores y peores.

Saludos.

8
Foro general / Re: Encontré un número perfecto impar
« en: Hoy a las 10:18 am »
Hola

 danizafa: Bienvenido al foro.

 Recuerda leer y seguir  las reglas del mismo así como el tutorial del LaTeX para escribir las fórmulas matemáticas correctamente.

Cumple las mismas condiciones que todos los números perfectos. A saber:

1) Es igual a la suma de sus divisores, sin tener en cuenta a él mismo.

El único divisor del \( 1 \) es el \( 1 \); pero no es un divisor propio, por es "él mismo" como tu dices. Por tanto el \( 1 \) NO tiene divisores propios. Como no tiene divisores propios la suma de ellos es \( 0\neq 1 \) y por tanto el \( 1 \) no es un número perfecto.

Citar
4) Además reafirma que el número 1 es el número primo por excelencia

El número uno NO es primo con la definición usual de primo que se usa en matemáticas; otra cosa es que cambies la definición y lo consideres primo. Sea como sea como apunta Fernando un debate sobre este punto sería una pérdida de tiempo: es pura cuestión de nombres y convenio. Si puede decirse algo sobre el porqué de esos convenios, aunque sería otra historia; si te interesa el tema busco algún hilo en el foro donde ya se haya tratado (lo hay, si mal no recuerdo).

1- Un número perfecto es un número triangular, lo que es igual a la sumatoria desde k = 1 hasta n de k = n(n+1)/2

Cada uno de esos valores de k es un factor del número perfecto.

desde k = 2 en adelante, es imposible encontrar un número perfecto impar debido a que el 2 es un factor para cualquier número perfecto. Lo que quiere decir que todos los números perfectos son divisibles por 2, por lo que TODOS LOS NÚMEROS PERFECTOS MAYORES A 1, SON PARES.


No acabo de entender lo que haces. En primer lugar está demostrado que todo número perfecto PAR es triangula; pero no se sabe si todo número perfecto impar (en caso de que existiese) es triangular.

En segundo lugar un número triangular es de la forma \( n(n+1)/2 \) y ese cociente en principio pude ser par o impar. Así que incluso usando (sólo) que un número perfecto es triangular de ahí no se deduce que sea impar.

Citar
simplemente queria compartir mis analisis... Pero veo que no son bienvenidos

Eres bienvenido en el foro.

Saludos.

P.D. También podría ser que en la línea que apunta Fernando estés considerando otra definición distinta de número perfecto y con ella \( 1 \) si lo sea. En ese sentido deberías de reflexionar: ¿realmente crees que si con la definición usual el \( 1 \) fuese considerado perfecto la cuestión de la existencia de un número perfecto impar permanecería como un problema abierto?  ;)

9
Álgebra y Aritmética Básicas / Re: Duda con vectores
« en: Ayer a las 10:15 pm »
Hola

Entonces, ¿qué criterio tengo que tomar a la hora de elegir las bases que mencionas?

Ninguno en especial, porque el resultado va a ser el mismo. La única forma de descomponer \( (3,5) \) como suma de un vector paralelo a \( (1,2) \) y otro perpendicular a él es:

\( (3,5)=(13/5,26/5)+(2/5,-1/5) \)

Independientemente de la elección de las bases que hagas.

Saludos.


10
Hola

En el conjunto de los números enteros \( Z \) se define la siguiente topología:
\( T=\{{A\subset{Z}/2n\in{A}\Longleftrightarrow{2n-1}\in{A}}, \forall{n\in{Z}}\} \)

No me queda claro cómo se interpretaría esto. Es decir no me queda claro qué característica deben poseer los conjuntos para que en esta topología sean abiertos, porque en la definición dice que la condición de que tanto un número par como su anterior deben pertencer al conjunto, pero "\( \forall{n\in{Z}} \)", lo que hace pensar que el único conjunto que puede cumplir esto sería el propio connjunto de los enteros \( Z \). Leído coloquialmente diría que: "Un subconjunto de los enteros es abierto si contiene a todos los números pares e impares". Porque si se hubiera referido a que \( \exists{n\in{Z}/2n\in{Z}\wedge2n-1\in{Z}} \) no debería aparecer el símbolo de "para todo" y, por ejemplo, el conjunto \( A=\{4;3\} \) sería abierto.
Pero no me queda claro cómo interpretarlo.

Significa que \( T \) está formado por los conjuntos \( A \) que cumplen la siguiente propiedad:

Si \( 2n\in A \) entonces \( 2n-1\in A \)

Es decir si un número par está en \( A \) entonces también está su número impar inmediatamente anterior; pero eso no obliga a que estén todos los pares.

O por oposición: no es abierto el conjunto contiene a un número par pero no a su impar inmediatamente anterior.

Por ejemplo serían abiertos:

\( \emptyset, \{3,4\},\{7\},\{1,3,5,6\},\{11,12,19,20,101,117,119\} \)

(el vacío también; en caso contrario debería de contener un par pero no su impar anterior y no es así simplemente porque no contiene ningún elemento par).

No lo serian:

\( \{1,3,18\},\{6\},\{4,5,6,7,8\},\{12\} \)

Es topología:

1) Claramente el vacío y el total están en \( T \).
2) Si \( A,B\in T \) entonces \( A\cap B\in T \).

Efectivamente, si \( 2n\in A\cap B \), entonces:

- \( 2n\in A \) y como \( A\in T \) entonces \( 2n-1\in A \).
- \( 2n\in B \) y como \( B\in T \) entonces \( 2n-1\in B \).

y por tanto \( 2n-1\in A\cap B \).

3) Si \( \{A_i\}_{i\in I}\subset T \) entonces \( \displaystyle\bigcup_{i\in T}A_i\in T \).

Efectivamente, si \( 2n\in \displaystyle\bigcup_{i\in T}A_i \), entonces \( 2n\in A_{i_0} \) para algún \( i_0\in I \). Como \( A_{i_0}\in T \) entonces \( 2n-1\in A_{i_0} \) y \( 2n-1\in\displaystyle\bigcup_{i\in T}A_i \).

Saludos.

11
Hola

Primero, menciona que no es distancia... porque no lo es? que es lo que falla , ya que debe cumplir 3 propiedades. Cual no?
No es distancia por que te he dado dos funciones, \( f,g \) que son diferentes pero su distancia es cero

Recalcando y subrayando esto. Juan Pablo Sancho te ha dados dos funciones \( f[Tex] y [tex]g \) tales que \( d(f,g)=0 \) pero sin embargo \( f\neq g \).

Tales funciones son:

\( f(x)=\begin{cases}{1}&\text{si}& x=a\\0 & \text{si}& x\in (a,b]\end{cases} \)

\( g(x)=\begin{cases}{-1}&\text{si}& x=a\\0 & \text{si}& x\in (a,b]\end{cases} \)

Por tanto falla la propiedad (1).

Saludos.

12
Hola

¿Puede qué aquí también haya erratas similares?

Spoiler
[cerrar]

Similares no, pero ERRORES si.

Está definición está MAL:

Citar
\( A=\{u_1;u_2;\ldots;u_n\}\subset V \) es linealmente independiente si \( \vec 0=\alpha_1\cdot u_1+\alpha_2\cdot u_2+\ldots+\alpha_n\cdot u_n \) y \( \exists i:\alpha_i=0 \)

Sería:

\( A=\{u_1;u_2;\ldots;u_n\}\subset V \) es linealmente independiente si

 \( \vec 0=\alpha_1\cdot u_1+\alpha_2\cdot u_2+\ldots+\alpha_n\cdot u_n\quad \Rightarrow\quad\alpha_i=0 \) para todo \( i\in \{1,2,\ldots,n\} \)

Tampoco está muy bien redactada la definición de linelamente dependientes.

Debería de ser: es linealmente dependiente si y sólo si existe alguna combinación lineal  \( \alpha_1\cdot u_1+\alpha_2\cdot u_2+\ldots+\alpha_n\cdot u_n \) igual al vector cero con algún \( \alpha_i\neq 0 \).

Los ejemplos están casi bien. No obstante al resolver los sistemas debería de poner una equivalencia y no un implica y además al segundo le falta un signo menos:

\( \left\{\begin{array}{l}{\alpha_1+2\alpha_2=0}\\ {2\alpha_1+4\alpha_2=0}\\\end{array}\right.\quad\color{red} \Longleftrightarrow{}\quad \alpha_1=-2\alpha_2\color{black} \)

en lugar de:

\( \left\{\begin{array}{l}{\alpha_1+2\alpha_2=0}\\ {2\alpha_1+4\alpha_2=0}\\\end{array}\right.\quad\color{red} \Longrightarrow{}\quad \alpha_1=2\alpha_2\color{black} \)

Saludos.

13
Hola

Hola, no entiendo como llego a lo que esta marcado en rojo, el resto lo entiendo.




En principio simplemente habría resuelto el sistema de ecuaciones:

\( \alpha_1+2\alpha_2=x_1 \)
\( 2\alpha_1+3\alpha_2=x_2 \)

Lo que pasa es que ese texto tiene muchas erratas.

1) La solución correcta del sistema que está escrito es:

\( \alpha_1=-3x_1+2x_2,\qquad \alpha_2=2x_1-x_2 \)

2) En realidad el sistema no debería de ser ese. Dado que los vectores eran \( (1,1) \) y \( (2,3) \), tendrías:

\( (x_1,x_2)=\alpha_1(1,1)+\alpha_2(2,3)  \)

y de ahí el sistema:

\( \alpha_1+2\alpha_2=x_1 \)
\( \color{red}\alpha_1\color{black}+3\alpha_2=x_2 \)

cuya solución sería ahora:

\( \alpha_1=3x_1-2x_2,\qquad \alpha_2=-x_1+x_2 \)

Que el cúmulo de erratas del texto no te despiste de las ideas que hay detrás.

Saludos.

14
Triángulos / Re: Cuadrados con vértices colineales
« en: Ayer a las 10:00 am »
Hola

No me cierra que el ángulo sea constante si se varía la longitud de los lados de los cuadrados, que no esta indicada en el enunciado, No he hecho cuentas pero...¿ a que se debería su constancia?

No hay que hacer cuentas; fíjate que el lado del cuadrado \( AB\color{red}C\color{black}D \) queda determinado por el lado del lado \( EFG\color{red}C\color{black} \). Remarco en rojo que tiene un vértice común.

- Dibuja el cuadrado \( EFGC \).
- Dibuja la diagonal del cuadrado \( GE \).
- Sobre esa diagonal \( GE \) mide la misma distancia \( GE \) y te da el punto \( D \).
- Y listo \( CD \) es la arista del cuadrado \( ABCD \) totalmente determinada.

Gracias por tu ayuda... pero busqué la pregunta original y faltaba un detalle importante..se menciona que el punto E es exterior, al cuadrado ABCD, entonces creo que el dibujo quedaría así.

Es simplemente considerar entonces el cuadrado verde de mi dibujo, hacia el otro lado. Con los mismos cálculos que he hecho:

\( \widehat{EA'D}=180^o-\widehat{A'ED}-\widehat{EDA'}=180^o-45^o-90^o-\widehat{CDE}=45^o-18.43^o=26.57^o \)

Saludos.

15
Probabilidad / Re: Ejercicio de probabilidad condicional
« en: Ayer a las 08:50 am »
Hola

GPT dijo:

Chat GPT: NO es fiable en absoluto para cuestiones sobre matemáticas. Entonces puede decir "cualquier cosa".

delmar ha explicado el razonamiento.

 -El mazo tiene \( 52 \) cartas. ¿De cuántas formas distintas podemos sacar tres de ellas?.

  Tenemos \( 52 \) opciones para la primera de carta. Por cada una de ellas, para la segunda carta quedan \( 52-1=51 \) opciones: no puede repetirse la carta que salió de primera. Y finalmente para la tercera carta \( 52-2=50 \) opciones. Por tanto los casos totales son:

 \( 52\cdot 51\cdot 50 \).

 -El mazo tiene cuatro ases. ¿De cuántas formas podemos sacar tres de ellos?.

 Pues por un razonamiento análogo \( 4 \) opciones para el primero; \( 4-1=3 \) para el segundo y \( 4-2=2 \) para el tercero. En total:

 \( 4\cdot 3\cdot 2 \)

 Por tanto la probabilidad pedida:

\( \dfrac{4\cdot 3\cdot 2}{52\cdot 51\cdot 50} \).

Saludos.

16
Hola

Profesor no entiendo lo siguiente queda así?

Fila 3 = Fila 3 - (2.3/ 5.5) * Fila 2
La matriz aumentada después de este paso es:

\( \left(\begin{array}{rrr|r}
4&-1&1&8\\
0&5.5&\color{red}2\color{black}&\color{red}-2\color{black}\\
0&0&3&0.9\\\end{array}\right) \)

   \( x_3= -0.09 / 3 = -0.03
 \)
\(    x_2 = (-2 - (2 * (-0.03))) / 5.5 ≈ -1 \)

\(    x_1 = (8 + (-1) + (1 * (-0.03))) / 4 ≈ 1
 \) no entiendo, no me coincide el resultado con el que proporcionan en el ejercicio profesor

 Es que ya has cambiado la segunda fila. ¿Por qué pones los términos que he marcado en rojo? No están bien. Recuerda que se obtenían restando a la segunda fila original \( 1/4 \) de la primera, te lo detalle en mi primera respuesta. De manera que llegaríamos a:

\( \left(\begin{array}{rrr|r} 4&-1&1&8\\ 0&5.5&1.5&-1\\ 0&2.3&3.8&9\\\end{array}\right) \)

 Ahora tenemos que hacer un cero donde está el \( 2.3 \) usando el \( 5.5 \). Para ello a la tercera fila le sumamos la segunda dividirá por \( 5.5 \) y multiplicada por \( -2.3 \):

 Tienes \( 1.5/5.5=0.2727\ldots\approx 0.27 \). Y \( 0.27\cdot 2.3=0.621\approx 0.62 \). Después \( 3.8-0.62=3.18\approx 3.2 \).

 Análogamente tenemos que hacer \( 9+(-2.3)\cdot (-1/5.5) \) redondeando en cada paso a dos cifras significativas. Quedará \( 9.4 \).

 Llegamos a:

\( \left(\begin{array}{rrr|r} 4&-1&1&8\\ 0&5.5&1.5&-1\\ 0&0&3.1&9.4\\\end{array}\right) \)

 Entonces de la última ecuación:

 \( 3.1x_3=9.4 \) y \( x_3=9.4/3.1=3.0322\ldots \approx 3 \).

 De la anterior:

 \( 5.5x_2+1.5x_3=-1 \) y \( x_2=(-1-1.5\cdot x_3)/5.5=-5.5/5.5=-1 \)

 De la primera:

\( 4x_1-x_2+x_3=8 \) y \( x_1=(8+x_2-x_3)/4=4/4=1 \)

Saludos.

17
Triángulos / Re: Cuadrados con vértices colineales
« en: 02 Mayo, 2024, 09:21 am »
Hola

 Pie: pero nota que los dos cuadrados tienen un vértice común \( C \).

 Yo diría que el dibujo es así. Con dos posibilidades para el cuadrado \( ABCD \). Me decanto por el cuadrado rojo porque así los vértices están ordenados en ambos cuadrados con el mismo sentido (contrario a las agujas del reloj).


 Sin pérdida de generalidad se puede suponer que el lado de \( EFGC \) es \( 1 \) y por tanto su diagonal \( \sqrt{2} \).

 En el triángulo \( CDE \) por el Teorema del Coseno:

 \( CD^2=CE^2+DE^2-2\cdot CD\cdot DE\cdot cos(135^o)=1+2+2\cdot \sqrt{2}\cdot \dfrac{\sqrt{2}}{2}=5\quad \Rightarrow{}\quad CD=\sqrt{5} \)

 Por el Teorema de los Senos en \( CDE \):

\(  sin(\widehat{CDE})=\dfrac{CE\cdot sin(135^o)}{CD}=\dfrac{\sqrt{10}}{10} \)   (*)

 Por el Teorema del Coseno en \( EDA \):

\( EA^2=ED^2+AD^2-2\cdot ED\cdot AD\cdot cos(\widehat{EDA})=2+5-2\sqrt{2}\sqrt{5}\cdot sin(\widehat{CDE})=5=AD^2 \)

 Por tanto el tríangulo \( EAD \) es isósceles y:

\( \widehat{EAD}=180^o-2\widehat{EDA}=2(90^o-\widehat{EDA})=2\cdot \widehat{CDE} \)

 De (*) se tiene que \(  \widehat{CDE}=arcsin(\sqrt{10}/10)\approx 18.43^o \)

y así

 \( \widehat{EAD}=2\cdot 18.43^o=36.86^o \)

 ó sin perder precisión:

\(  cos(\widehat{EAD})=cos(2\cdot \widehat{CDE})=1-2sin^2(\widehat{CDE})=1-2\cdot \dfrac{1}{10}=\dfrac{4}{5} \)

 Es decir:

 \( \widehat{EAD}=arccos(4/5) \).

Saludos.

18
Foro general / Re: Ejercicios de diagonalización
« en: 01 Mayo, 2024, 09:07 pm »
Hola

 Por ampliar un poco más el segundo caso, comprueba que los autovalores son \( 1,1,c \) lo cuál te lleva a distinguir dos casos:

- Si \( c\neq 1 \) hay dos autovalores:

\( \lambda_1=1 \) con multiplicidad algebraica \( 2 \).
\( \lambda_2=c \) con multiplicidad algebraica \( 1 \).

 La suma de algebraicas coincide con el tamaño de la matriz (o si quieres todos los autovalores son reales) y por tanto siempre triangulariza (de hecho la matriz original es triangular). Diagonaliza si además las multiplicidades geométricas y algebraicas de cada autovalor coinciden. Si la algebraica es uno, al geométrica también; así que sólo tienes que comprobar la geométrica para \( \lambda_1=1 \)

\( mg(1)=3-rango(A-1\cdot Id) \)

 Termina...

- Si \( c=1 \) hay un autovalor:

\( \lambda_1=1 \) con multiplicidad algebraica \( 3 \).

 La suma de algebraicas coincide con el tamaño de la matriz (o si quieres todos los autovalores son reales) y por tanto siempre triangulariza (de hecho la matriz original es triangular). Ahora tienes que comprobar si la multiplicidad geométrica coincide con la algebraica:

\( mg(1)=3-rango(A-1\cdot Id) \)

 Termina...

Saludos.

19
Hola

He comprendido que si hago \( 0\leq \theta\leq 2\pi \) estaría dando dos vueltas, pero por qué no valdría el intervalo \( 0\leq \theta\leq \pi \)?, dado que al ser \( r=2\cos\theta \) cuando el coseno es negativo \( r \) sería negativa.

\( r \) no puede ser negativa en las coordenadas cilíndricas. Entonces que  \( r=2\cos\theta\geq 0 \) es lo que restringe el ángulo a los valores en los cuáles el coseno es positivo.

Saludos.

20
Métodos Numéricos / Re: Polinomio de Lagrange
« en: 01 Mayo, 2024, 10:40 am »
Hola

Suponga que \( P_n \) es un polinomio de grado \( \leq{n} \) que interpola a la función \( f(x)=e^{\displaystyle\frac{x-2}{2}} \) en los nodos
\( 0=x_0<...<x_n=2 \) con \( x_i=2\displaystyle\frac{i}{n} \) para \( i=0,...,n \).
Construya el polinomio \( Pn \)

El nodo cero, es 0, pero me estoy haciendo lio para poder hallar los demás (nodo uno y nodo dos); alguna ayuda/sugerencia.

¿Pero te refieres a calcular los \( x_i \)?.

\( x_0=2\cdot \dfrac{0}{n}=0 \)

\( x_1=2\cdot \dfrac{1}{n}=\dfrac{2}{n} \)

\( x_2=2\cdot \dfrac{2}{n}=\dfrac{4}{n} \)

\( x_3=2\cdot \dfrac{3}{n}=\dfrac{6}{n} \)

\( \ldots \)

Evidentemente si no fijas \( n \) lo le puedes dar un valor concreto.

Saludos.

Páginas: [1] 2 3 4 ... 2744